抱歉,您的浏览器无法访问本站
本页面需要浏览器支持(启用)JavaScript
了解详情 >

一方の笔记本

The only source of knowledge is experience.

这篇博客用于记录一些杂七杂八的数学题目以及相关的知识。

\(a_n = \displaystyle (1+\frac{1}{n})^n\),求证:\(\forall n \in \text{N}^+, a_{n+1}>a_n\)

解法一

\(a_n=\displaystyle \prod_{i=1}^{n} (1 +\frac{1}{n})=\prod_{i=1}^{n} (1 +\frac{1}{n})\cdot 1 \leq [\frac{\displaystyle \sum_{i=1}^{n}(1+\frac{1}{n})+1}{n+1}]^{n+1}=(1+\frac{1}{n+1})^{n+1}=a_{n+1}\)

上式中的取等条件为\(\displaystyle \frac{1}{n}+1=1\),这显然是不可能成立的,故\(a_{n+1} > a_{n}\)

这个解法使用了均值不等式,即\(\forall a_1, a_2,\dots,a_n \in \text{R}\)\(H_n \leq G_n \leq A_n\leq Q_n\),其中四个均值分别为调和平均数、几何平均数、算数平均数和平方平均数,不等式取等条件都是\(\forall i,j\in\{1,2,\dots,n\},a_i=a_j\)。上面的证明使用到了\(G_n\leq A_n\)的变形,即\(\displaystyle \prod_{i=1}^{n}a_i \leq (\frac{\displaystyle \sum_{i=1}^{n}a_i}{n})^n\)

解法二

\(\displaystyle \frac{a_{n+1}}{a_n} = [1-\frac{1}{(n+1)^2}]^n \cdot \frac{n+2}{n+1} \geq [1-\frac{n}{(n+1)^2}] \cdot \frac{n+2}{n+1} > (1-\frac{1}{n+2}) \cdot \frac{n+2}{n+1}=1\)

以上证明使用了伯努利不等式,即\(\forall x > -1\)

  • \(n\geq1 \Rightarrow (1+x)^n\geq1+nx\)
  • \(n\in[0,1] \Rightarrow(1+x)^n\leq1+nx\)

此外,\(\displaystyle \frac{n}{(n+1)^2}=\frac{1}{n+\displaystyle\frac{1}{n}+2}<\frac{1}{n+2}\)

解法三

\(y=e^{\displaystyle x\ln(1+\frac{1}{x})}, x\in \text{R}^+\),则\(f'(x)=[\displaystyle\ln(1+\frac{1}{x})-\frac{1}{x+1}]f(x)\)

\(g(x)=\ln(1+x)-\displaystyle\frac{x}{1+x}\),则\(g'(x)=\displaystyle\frac{x}{1+x} > 0\),那么\(f'(x)>0\),那么\(f(x)\)\(\text{R}^+\)上单调递增,原命题显然。

删数问题

题目描述

已知有一数组,其元素为\(1,2,3,\dots,100\)\(100\)个数,现随机删除\(2\)个数,并将其余\(98\)个数乱序存于另一数组,试问如何求出这\(2\)个数?

求解思路

对于删除\(1\)个数的情况,可以利用\(100\)个数的总和为\(5050\)的条件,遍历剩余的\(99\)个数并得到总和\(B_1\),则被删去的数就是\((5050 -B_1)\)

事实上,不难发现原来\(100\)个数的乘积也是常数\(100!\),那么可以得到剩下\(98\)个数的和、乘积,即可得到被删除的\(2\)个数的和、乘积,逆用韦达定理即可构造关于这\(2\)个数的一元二次方程,求解这个方程,得到的两根一定是被删去的数。

推广

那么,以上求解方程的思想是否可以推广到删除超过\(2\)个数的情况呢?我经过思考后认为可以。

设被删除的数为\(\{b_1,b_2,\dots,b_t\}\)\(t\)个,可以通过构造一元\(t\)次方程求解被删除的数,其中\(1\leq t \leq99\)

首先可以通过剩余\((100-t)\)个数得到\(\displaystyle \sum_{i=1}^tb_i\)\(\displaystyle \prod_{i=1}^tb_i\),不妨分别记之为\(B_1\)\(B_t\)

考虑\(\displaystyle (\sum_{i=1}^{100}i)^2 = 5050^2\),那么:

\[\begin{aligned} 5050^2 & =\displaystyle (\sum_{i=1}^{100}i)^2 \\ &= ((\sum_{i=1}^tb_i)+(5050-\sum_{i=1}^tb_i))^2\\ & = \displaystyle (\sum_{i=1}^tb_i)^2 + (5050-\sum_{i=1}^tb_i)^2+2(\sum_{i=1}^tb_i)(5050-\sum_{i=1}^tb_i) \\ & = B_1^2+(5050-B_1)^2+10100B_1+2\displaystyle (\sum_{i=1}^tb_i)(\sum_{i=1}^tb_i) \\ &= B_1^2+(5050-B_1)^2+10100B_1+2\displaystyle (\sum_{i=1}^tb_i^2 + \sum_{1\leq i < j \leq t} b_i b_j)\end{aligned}\]

不妨记\(\displaystyle \sum_{1\leq i < j \leq t} b_i b_j = B_2\)。由于\(100\)个数的平方和是一定的,那么可以得到\(\displaystyle \sum_{i=1}^tb_i^2\)也是常数。恒等式左侧中仅有\(B_2\)是未知数,而右侧为常数,那么可以求解得到\(B_2\)

现在不妨观察已经得到的\(3\)个常系数,这\(3\)个系数与\(\{b_1,b_2,\dots,b_t\}\)之间的关系为:

\[\begin{cases} \displaystyle \sum_{i=1}^{t}b_i=B_1\\ \displaystyle \sum_{1\leq i < j \leq t} b_i b_j = B_2\\b_1b_2\cdots b_t = B_t\\ \end{cases}\]

不难发现以上等式左侧都是关于\(b1,b_2,\dots,b_t\)的轮换对称式,那么可以类比求解\(B_2\)的方法,利用\(\displaystyle (\sum_{i=1}^{100}i)^3 = 5050^3\),得到\(\displaystyle \sum_{1\leq i < j <k \leq t} b_i b_j b_k = B_3\),最终可以得到\(B_1,B_2,\dots,B_t\)\(t\)个常数。上述推理的简单证明如下:

  • 归纳奠基:显然通过\(\displaystyle(\sum_{i=1}^{100}i)^1=5050\)已经求得\(B_1\)
  • 归纳推理:设\(k\geq2\),已知\(\displaystyle (\sum_{i=1}^ti)^k=5050^k\)以及\(B_1,B_2,\dots,B_{k-1}\),而\(\displaystyle ((\sum_{i=1}^tb_i)+(5050-\sum_{i=1}^tb_i))^k\)的的展开式可由二项式定理求得为\(\displaystyle \sum_{j=0}^k C_k^j(\sum_{i=1}^tb_i)^{j}(5050-\sum_{i=1}^tb_i)^{k-j}\),进一步展开后\((k+1)\)项中除了\(B_k\)未知之外,其余各项均已求得,因此可以求得\(B_k\)

利用构造出的\(t\)个关于\(b_1,b_2,\dots,b_t\)即可根据系数构造一元\(t\)次方程:

\[\displaystyle \prod_{i=1}^t(x-b_i)=x^t-B_1x^{t-1}+\cdots+(-1)^tB_t=0\]

求解这个方程就可以得到被删除的\(t\)个数。

卡特兰数

卡特兰数的其中一个应用为:数\(1 \sim n\)按序入栈,求课内的出栈序列的个数。

卡特兰数的通项公式为\(C_n = \frac{C_{2n}^n}{n+1},n\in \text{N}^+\)

\(I=\lim\limits_{x \to 1^-} (1-x)^3 \displaystyle \sum_{n=1}^{\infty} n^2x^n\)

首先需要知道以下公式。 \[\displaystyle\sum_{n=0}^{\infty}x^n = \frac{1}{1-x}, x\in(-1,1)\]

对上式两端求导得到如下公式。 \[\displaystyle\sum_{n=0}^{\infty}(n+1)x^{n} = \frac{1}{(1-x)^2}, x\in(-1,1)\] \[\displaystyle\sum_{n=0}^{\infty}(n+1)(n+2)x^{n} = \frac{2}{(1-x)^3}, x\in(-1,1)\]

\(n^2=(n+1)(n+2)-3(n+1)+1\)可以得到如下的结果。 \[\displaystyle \sum_{n=1}^{\infty} n^2x^n = \frac{2}{(1-x)^3} - \frac{3}{(1-x)^2} + \frac{1}{1-x} , x \in(-1,1)\]

最终可以得到\(I=2\)

无穷级数中的下标变化问题

在前面求解过程中涉及到了两次求导,事实上隐含了两次下标变换,具体如下所示。

\[(\frac{1}{1-x})' = \displaystyle\sum_{n=1}^{\infty}nx^{n-1} = \displaystyle\sum_{n-1=0}^{\infty}[(n-1)+1]x^{n-1} = \displaystyle\sum_{n=0}^{\infty}(n+1)x^{n}, x\in(-1,1)\] \[[\frac{1}{(1-x)^2}]'= \sum_{n=1}^{\infty}n(n+1)x^{n-1}= \sum_{n-1=0}^{\infty}[(n-1)+1][(n-1)+2]x^{n-1} = \displaystyle\sum_{n=0}^{\infty}(n+1)(n+2)x^{n}, x\in(-1,1)\]

实际上,无穷级数的第一项需要分类讨论,因为对于\(f(x)=Cx^0\)\(f'(x) \neq Cx^{-1}\),而是\(f'(x)=0\),因此需要分类讨论。

无穷集合势相关的证明

证明:\([0,1]\sim (0,1)\)

可以构造如下的\([0,1] \to (0,1)\)的双射,证毕。

\[f(x) = \begin{cases} \displaystyle \frac{1}{2}, x = 1\\ \displaystyle \frac{1}{4}, x = 0\\ \displaystyle \frac{1}{4}x, x = \frac{1}{2^n}, n \in \{1,2,3,\dots\}\\ \displaystyle x, \text{else}\\ \end{cases}\]

证明:\(|\text{N}^+| < |(0,1)|\)

参考康托对角线证明。

\(I=\displaystyle \lim\limits_{n \to \infty} \sum_{i=1}^{n} \frac{1}{n+i}\)

解法一

可以使用定积分定义求解。

\[ I = \displaystyle \frac{1}{n}\sum_{i=1}^{n} \frac{1}{1+ \displaystyle\frac{i}{n}} = \int_{0}^{1}\frac{1}{1+x}dx=\ln 2\]

解法二

由不等式\(\displaystyle \frac{x}{1+x} < \ln(1+x) < x\),可以使用夹逼定理求解。

\[\displaystyle \frac{1}{n + 1} = \frac{\displaystyle \frac{1}{n}}{1 + \displaystyle \frac{1}{n}} < \ln(1 + \frac{1}{n}) =\ln(n+1) - \ln n < \frac{1}{n} \]

类推可以得到下列不等式。

\[\begin{cases} \displaystyle \frac{1}{n + 1} < \ln(n+1) - \ln n < \frac{1}{n}\\ \displaystyle \frac{1}{n + 2} < \ln(n+2) - \ln (n+1) < \frac{1}{n + 1}\\ \cdots\\ \displaystyle \frac{1}{2n} < \ln(2n) - \ln (2n-1) < \frac{1}{2n-1} \end{cases}\]

将以上\(n\)个不等式求和可得到如下不等式。

\[\displaystyle \sum_{i=1}^{n} \frac{1}{n+i} < \ln2 < \sum_{i=1}^{n} \frac{1}{n+i} + \frac{1}{n} - \frac{1}{2n} = \sum_{i=1}^{n} \frac{1}{n+i} + \frac{1}{2n}\]

进一步可知\(\displaystyle \ln2 - \frac{1}{2n} < \sum_{i=1}^{n} \frac{1}{n+i} < \ln2\),故\(I=\ln2\)

已知\(S=\{1,2,\dots,2000\}\),求满足条件\(S_i \subseteq S\)\(\displaystyle\sum_{e \in S_i}e \equiv 0 \pmod{5}\)的集合\(S_i\)的个数

题目来源于3Blue1Brown

不妨首先考虑\(S'= \{1,2,3,4,5\}\)以及生成函数\(g(x) = (1+x)(1+x^2)(1+x^3)(1+x^4)(1+x^5)\),考虑\(g(x)\)展开式中\(x\)各个幂次前对应的系数:注意展开等号右侧的过程,\(x\)的幂次实际上说明了在展开过程中在\(5\)个括号中选择的\(x\)的幂次之和,这就表明\(x^i\)的系数\(c_i\)可以代表\(S'\)和为\(i\)的集合的个数,其中\(i \in \{0,1,\dots,15\}\);同理,可以考虑生成函数 \(f(x) = \displaystyle \prod_{i=1}^{2000}(1+x^i) = \sum_{i=0}^{2000}c_ix^i\)

基于上述分析,原问题可转化为求解和式\(\displaystyle \sum_{i \equiv 0 \pmod{5}}c_i\)的值。记\(T_j = \displaystyle \sum_{i \equiv j \pmod{5}}c_i, j \in \{ 0,1,2,3,4\}\),那么下面需要求解\(T_0\)

注意到\(f(1)=2^{2000}\)\(f(-1)=0\),根据这两个式子可以得到\(f(x)\)展开式中\(x\)的奇数次项和偶数次项前的系数和都是\(2^{1999}\)。受此启发,引入\(\zeta = e^{\displaystyle\frac{2\pi}{5}i}\),计算\(f(\zeta^0)\)\(f(\zeta^1)\)\(f(\zeta^2)\)\(f(\zeta^3)\)\(f(\zeta^4)\),由复数性质可得如下结果。

\[\begin{cases} f(\zeta^0)=T_0+T_1+T_2+T_3+T_4=2^{2000}\\ f(\zeta^1)=T_0+\zeta^1 T_1+\zeta^2T_2+\zeta^3T_3+\zeta^4T_4=2^{400}\\ f(\zeta^2)=T_0+\zeta^2 T_1+\zeta^4T_2+\zeta^1T_3+\zeta^3T_4=2^{400}\\ f(\zeta^3)=T_0+\zeta^3 T_1+\zeta^1T_2+\zeta^4T_3+\zeta^2T_4=2^{400}\\ f(\zeta^4)=T_0+\zeta^4 T_1+\zeta^3T_2+\zeta^2T_3+\zeta^1T_4=2^{400} \end{cases}\]

\(f(1)\)显然可得。下面将主要求解\(f(\zeta)\),其余\(3\)个函数值同理可得,注意到\(\zeta\)\(z^5-1=0\)的根,因此有\(z^5-1=\displaystyle\prod_{i=0}^{4}(z-\zeta^i)\),令\(z=-1\),则\(\displaystyle\prod_{i=0}^{4}(1+\zeta^i)=2\),那么可以求解\(f(\zeta)\)如下所示。

\[f(\zeta)=[(1+\zeta^0)(1+\zeta^1)(1+\zeta^2)(1+\zeta^3)(1+\zeta^4)]^{400}=2^{400}\]

\(\displaystyle\sum_{i=0}^4\zeta^i=0\),对以上五个式子求和最终可得\(T_0=\displaystyle \frac{1}{5}(2^{2000} + 2^{400})\)

已知方阵\(A\)\(B\)满足关系\(AB=A+B\),求证:\(AB=BA\)

\[AB=A+B \Rightarrow (A-E)(B-E)=E \Rightarrow (A-E)^{-1} = (B-E) \Rightarrow (A-E)(B-E) = (B-E)(A-E) \Rightarrow AB = BA\]

群论相关的一些证明

我比较喜欢这个课程

已知\(<V,\cdot>\)为半群,且\(\forall a,b \in V, a \neq b \Rightarrow ab \neq ba\),求证以下命题

  • (1)\(V\)中幂等律成立
  • (2)\(\forall a,b \in V, aba = a\)
  • (3)\(\forall a,b,c \in V, abc = ac\)

(1)\(\forall a \in V\),假定\(a \cdot a \neq a\),那么\(a \cdot (a \cdot a) \neq (a \cdot a) \cdot a\),对于满足结合律的半群而言产生矛盾,因此\(V\)中幂等律成立。

(2)\(\forall a,b \in V\),假定\(aba \neq a\),那么\(aba \cdot a \neq a \cdot aba\),根据结合律和幂等律可知\(aba \neq aba\),产生矛盾,因此命题成立。

(3)\(\forall a,b,c \in V\),假定\(abc \neq ac\),那么\(abc \cdot ac \neq ac \cdot abc\),由结合律和(2)可知\(abc \neq abc\),产生矛盾,因此命题成立。

抽象函数相关的题目

已知函数\(f(x)\)\(\text{R}\)可导,且\(f(x+y)=e^yf(x)+e^xf(y)\)以及\(f'(0)=2\),求\(f(x)\)的表达式

首先令\(x=y=0\),得到\(f(0)=0\)

再令\(y=\Delta x\),那么\(f(x+\Delta x) - f(x) = (e^{\Delta x} - 1)f(x)+e^xf(\Delta x)\)

由题目的\(f(x)\)处处可导及导数定义有: \[f'(x)=\lim_\limits{\Delta \to 0} \displaystyle \frac{f(x + \Delta x) - f(x)}{\Delta x}=f(x)\lim_\limits{\Delta \to 0}\frac { e^{\Delta x} - 1}{\Delta x} + e^x \lim_\limits{\Delta x \to 0} \frac{f(\Delta x) - f(0)}{\Delta x - 0}=f(x)+2e^x\]

求解微分方程得到\(f(x)=e^x(C+2x),C\in \text{R}\)

对于微分方程\(\displaystyle \frac{dy}{dx} + P(x)y=Q(x)\),其通解为\(y=e^{-\displaystyle \int P(x) dx}(C + \displaystyle \int e^{P(x)}Q(x)dx)\)

已知\(A\)是实反对称矩阵,求证:\(B=(E-A)(E+A)^{-1}\)为正交矩阵

\[B^TB=[(E-A)(E+A)^{-1}]^T(E-A)(E+A)^{-1} = (E-A)^{-1}(E+A)(E-A)(E+A)^{-1}\]

注意到\((E-A)(E+A)=E-A^2=(E+A)(E-A)\),因此\(B^TB=E\),证毕。

事实上,只要\(B=\displaystyle\prod_{i} f_i(A)\),其中\(A\)为方阵,那么\(f_{i_1}(A)\)\(f_{i_2}(A)\)就可以交换乘积顺序。

求曲线\(x+y+e^y=0\)的渐近线

显然曲线\(x=-(e^y+y)\)只有斜渐近线,设之方程为\(x = ay+b\)

\[a = \lim_\limits{y \to \infty} \displaystyle \frac{x}{y} = -1 - \lim_\limits{y \to \infty}\frac{e^y}{y}\]

以上极限仅在\(y\to -\infty\)时存在,并且\(a = -1\)

\[b = \lim_\limits{y \to -\infty} \frac{x - ay}{y} = 0\]

因此所求渐近线方程为\(x+y=0\)通过本题可知,不要局限地将任何惯用的因(自)变量符号默认当作因(自)变量。

评论